2017年入試問題研究に戻る

京大特色1番解答

複素数平面で考え,点 $ \mathrm{P}_n $ に対応する複素数を $ z_n $ とおく. $ z_1=1,\ z_2=i $ である. また複素数 $ \alpha $ を $ \alpha=\cos(\pi r)+i\sin(\pi r) $ とする. $ \alpha\bar{\alpha}=1 $ で, $ \alpha+\bar{\alpha}=2\cos(\pi r) $ である. このとき漸化式は \[ z_{n+2}=(\alpha+\bar{\alpha})z_{n+1}-\alpha\bar{\alpha}z_n \] となる.これより \[ z_{n+2}-\alpha z_{n+1}=\bar{\alpha}\left(z_{n+1}-\alpha z_n \right) \] となり \[ z_{n+1}-\alpha z_n =\bar{\alpha}^{n-1}(z_2-\alpha z_1)=\bar{\alpha}^{n-1}(i-\alpha) \] となる.同様に \[ z_{n+1}-\bar{\alpha} z_n =\alpha^{n-1}(z_2-\bar{\alpha} z_1)=\alpha^{n-1}(i-\bar{\alpha}) \] も成り立つ.辺々引いて \begin{eqnarray*} \left(\alpha-\bar{\alpha} \right)z_n&=&\alpha^{n-1}(i-\bar{\alpha})-\bar{\alpha}^{n-1}(i-\alpha)\\ &=&\left(\alpha^{n-1}-\bar{\alpha}^{n-1}\right)i-\left(\alpha^{n-2}-\bar{\alpha}^{n-2} \right) \end{eqnarray*} よって \[ 2i\sin(\pi r)z_n=2i\sin((n-1)\pi r)i-2i\sin((n-2)\pi r) \] より \[ z_n=\dfrac{\sin((n-1)\pi r)i-\sin((n-2)\pi r)}{\sin(\pi r)} \] したがって条件 $ \left| \overrightarrow{\mathrm{OP}_n}\right|\geqq 1 $ は, $ |z_n|^2\geqq 1 $ ,つまり \[ \sin^2((n-1)\pi r)+\sin^2((n-2)\pi r)\geqq \sin^2(\pi r) \] となる.これより \[ 1-\cos((2n-2)\pi r)+1-\cos((2n-4)\pi r)\geqq 1-\cos(2\pi r) \] となる. \[ \cos((2n-2)\pi r)+\cos((2n-4)\pi r)=2\cos((2n-3)\pi r) \cos(\pi r) \] なので, \[ 1-2\cos(2n-3)\pi r \cos \pi r\geqq -\cos(2\pi r)=1-2\cos^2\pi r \] つまり \[ \cos((2n-3)\pi r) \cos (\pi r)\le\cos^2(\pi r) \] である. $ r=\dfrac{1}{2} $ なら両辺0ですべての $ n $ で成立する. $ 0< r< \dfrac{1}{2} $ のとき $ 1>\cos \pi r>0 $ なので, \[ \cos((2n-3)\pi r) \le\cos(\pi r) \] となる. \[ \cos(\pi r)-\cos((2n-3)\pi r)=2\sin((n-1)\pi r)\sin((n-2)\pi r) \] なので, \[ \sin((n-1)\pi r)\sin((n-2)\pi r)\geqq 0 \] がすべての $ n $ で成立することが条件である.
$ r=\dfrac{q}{p} $ とおく. $ 0< r\leqq \dfrac{1}{2} $ より $ 2\leqq \dfrac{p}{q} $ である.
$ q\geqq 2 $ のとき. $ p $ を $ q $ で割った商を $ Q $ ,余りを $ R $ とする. $ p $ と $ q $ は互いに素なので, \[ \dfrac{p}{q}=Q+\dfrac{R}{q},\ 1\leqq R\leqq q-1 \] である.この結果, \[ Q< \dfrac{p}{q}< Q+1 \] なので, \[ \dfrac{qQ}{p}< 1< \dfrac{q(Q+1)}{p} \] となり, $ n-2=Q $ となる $ n $ をとると, \[ (n-2)\pi r< \pi< (n-1)\pi r \] となる.よって,このときは \[ \sin((n-1)\pi r)\sin((n-2)\pi r)< 0 \] となり,条件を満たさない $ n $ が存在する.
$ q=1 $ のとき. \[ (n-2)r=\dfrac{n-2}{p}< 整数=l< (n-1)r=\dfrac{n-1}{p} \] となる $ n $ は存在しない. もし存在すれば, \[ n-2< pl< n-1 \] となり,矛盾である. よってこの場合は,すべての $ n $ に対して条件が成立する.
条件を満たす $ r $ は \[ r=\dfrac{1}{p},\ \quad (p \geqq 2\ の整数) \] である.

問題